Difference between revisions of "2021 AMC 12A Problems/Problem 19"

(Video Solution by pi_is_3.14 (Using Triangle Inequality & Trigonometry))
(Solution: fixed problem with range of arcsin; only cos has the range)
Line 7: Line 7:
 
<math>\sin \left( \frac{\pi}2 \cos x\right)=\cos \left( \frac{\pi}2 \sin x\right)</math>
 
<math>\sin \left( \frac{\pi}2 \cos x\right)=\cos \left( \frac{\pi}2 \sin x\right)</math>
  
The interval is <math>[0,\pi]</math>, which is included in the range of both <math>\arccos</math> and <math>\arcsin</math>, so we can use them with no issues.
+
The interval is <math>[0,\pi]</math>, which is included in the range <math>\arccos</math>, so we can use it with no issues.
  
<math>\frac{\pi}2 \cos x=\arcsin \left( \cos \left( \frac{\pi}2 \sin x\right)\right)</math>
+
<math>\frac{\pi}2 \sin x=\arccos \left( \sin \left( \frac{\pi}2 \cos x\right)\right)</math>
  
<math>\frac{\pi}2 \cos x=\frac{\pi}2 - \frac{\pi}2 \sin x</math>
+
<math>\frac{\pi}2 \sin x=\frac{\pi}2 - \frac{\pi}2 \cos x</math>
  
<math>\cos x = 1 - \sin x</math>
+
<math>\sin x = 1 - \cos x</math>
  
 
<math>\cos x + \sin x = 1</math>
 
<math>\cos x + \sin x = 1</math>
  
This only happens at <math>x = 0, \frac{\pi}2</math> on the interval <math>[0,\pi]</math>, because one of <math>\sin</math> and <math>\cos</math> must be <math>1</math> and the other <math>0</math>. Therefore, the answer is <math>\boxed{C: 2}</math>
+
This only happens at <math>x = 0, \frac{\pi}2</math> on the interval <math>[0,\pi]</math>, because one of <math>\sin</math> and <math>\cos</math> must be <math>1</math> and the other <math>0</math>. Therefore, the answer is <math>\boxed{C) 2}</math>
  
 
~Tucker
 
~Tucker
 
  
 
== Video Solution by OmegaLearn (Using Triangle Inequality & Trigonometry) ==
 
== Video Solution by OmegaLearn (Using Triangle Inequality & Trigonometry) ==

Revision as of 14:08, 12 February 2021

Problem

How many solutions does the equation $\sin \left( \frac{\pi}2 \cos x\right)=\cos \left( \frac{\pi}2 \sin x\right)$ have in the closed interval $[0,\pi]$?

$\textbf{(A) }0 \qquad \textbf{(B) }1 \qquad \textbf{(C) }2 \qquad \textbf{(D) }3\qquad \textbf{(E) }4$

Solution

$\sin \left( \frac{\pi}2 \cos x\right)=\cos \left( \frac{\pi}2 \sin x\right)$

The interval is $[0,\pi]$, which is included in the range $\arccos$, so we can use it with no issues.

$\frac{\pi}2 \sin x=\arccos \left( \sin \left( \frac{\pi}2 \cos x\right)\right)$

$\frac{\pi}2 \sin x=\frac{\pi}2 - \frac{\pi}2 \cos x$

$\sin x = 1 - \cos x$

$\cos x + \sin x = 1$

This only happens at $x = 0, \frac{\pi}2$ on the interval $[0,\pi]$, because one of $\sin$ and $\cos$ must be $1$ and the other $0$. Therefore, the answer is $\boxed{C) 2}$

~Tucker

Video Solution by OmegaLearn (Using Triangle Inequality & Trigonometry)

https://youtu.be/wJxN1YPuyCg

~ pi_is_3.14

See also

2021 AMC 12A (ProblemsAnswer KeyResources)
Preceded by
Problem 18
Followed by
Problem 20
1 2 3 4 5 6 7 8 9 10 11 12 13 14 15 16 17 18 19 20 21 22 23 24 25
All AMC 12 Problems and Solutions

The problems on this page are copyrighted by the Mathematical Association of America's American Mathematics Competitions. AMC logo.png